How to solve $T(n) = T(2n/3) + lg^2 n$ by substitution?












1












$begingroup$


My solution through substitution is as follows:



$$T(n) = T(2n/3) + lg^2 (n)$$
$$T(2n/3) = T(4n/9) + lg^2 (2n/3)$$
$$T(4n/9) = T(8n/27) + lg^2 (4n/9)$$
And so on...



But my actual problem is how can I calculate the below step which cause to obtain order of the above expression:



$$lg^2 left(ncdot(2/3)ncdot(2/3)^2ncdot(2/3)^3ncdotsright).$$



Also I know the order is $theta(lg^3n)$.



Thanks!










share|cite|improve this question











$endgroup$












  • $begingroup$
    Hi, can you please add some brackets at the first 3 rows for the logarithms? It is abit confusing, but I guess it should be: $lg^2(2n/3)$. Also are you sure that the second row is $T(n)$ instead of $T(2n/3)$?
    $endgroup$
    – Zacky
    Dec 25 '18 at 13:11












  • $begingroup$
    @Zacky Hi! I've added those parentheses and You're right it's T(2n/3) instead of T(n).My bad!
    $endgroup$
    – Alireza_T
    Dec 25 '18 at 13:24












  • $begingroup$
    Also how do you end the recursion, I mean how did your teacher tought you to write afterwards $T(n)$ as a complete sum? I can answer you by how I've learnt, but it might differ.
    $endgroup$
    – Zacky
    Dec 25 '18 at 13:26












  • $begingroup$
    @Zacky My initial thought for this specific example is to reach T(1)! It's kinda like recursion tree problems to me with one branch!
    $endgroup$
    – Alireza_T
    Dec 25 '18 at 13:31


















1












$begingroup$


My solution through substitution is as follows:



$$T(n) = T(2n/3) + lg^2 (n)$$
$$T(2n/3) = T(4n/9) + lg^2 (2n/3)$$
$$T(4n/9) = T(8n/27) + lg^2 (4n/9)$$
And so on...



But my actual problem is how can I calculate the below step which cause to obtain order of the above expression:



$$lg^2 left(ncdot(2/3)ncdot(2/3)^2ncdot(2/3)^3ncdotsright).$$



Also I know the order is $theta(lg^3n)$.



Thanks!










share|cite|improve this question











$endgroup$












  • $begingroup$
    Hi, can you please add some brackets at the first 3 rows for the logarithms? It is abit confusing, but I guess it should be: $lg^2(2n/3)$. Also are you sure that the second row is $T(n)$ instead of $T(2n/3)$?
    $endgroup$
    – Zacky
    Dec 25 '18 at 13:11












  • $begingroup$
    @Zacky Hi! I've added those parentheses and You're right it's T(2n/3) instead of T(n).My bad!
    $endgroup$
    – Alireza_T
    Dec 25 '18 at 13:24












  • $begingroup$
    Also how do you end the recursion, I mean how did your teacher tought you to write afterwards $T(n)$ as a complete sum? I can answer you by how I've learnt, but it might differ.
    $endgroup$
    – Zacky
    Dec 25 '18 at 13:26












  • $begingroup$
    @Zacky My initial thought for this specific example is to reach T(1)! It's kinda like recursion tree problems to me with one branch!
    $endgroup$
    – Alireza_T
    Dec 25 '18 at 13:31
















1












1








1





$begingroup$


My solution through substitution is as follows:



$$T(n) = T(2n/3) + lg^2 (n)$$
$$T(2n/3) = T(4n/9) + lg^2 (2n/3)$$
$$T(4n/9) = T(8n/27) + lg^2 (4n/9)$$
And so on...



But my actual problem is how can I calculate the below step which cause to obtain order of the above expression:



$$lg^2 left(ncdot(2/3)ncdot(2/3)^2ncdot(2/3)^3ncdotsright).$$



Also I know the order is $theta(lg^3n)$.



Thanks!










share|cite|improve this question











$endgroup$




My solution through substitution is as follows:



$$T(n) = T(2n/3) + lg^2 (n)$$
$$T(2n/3) = T(4n/9) + lg^2 (2n/3)$$
$$T(4n/9) = T(8n/27) + lg^2 (4n/9)$$
And so on...



But my actual problem is how can I calculate the below step which cause to obtain order of the above expression:



$$lg^2 left(ncdot(2/3)ncdot(2/3)^2ncdot(2/3)^3ncdotsright).$$



Also I know the order is $theta(lg^3n)$.



Thanks!







notation asymptotics recursion substitution






share|cite|improve this question















share|cite|improve this question













share|cite|improve this question




share|cite|improve this question








edited Dec 26 '18 at 9:47







Alireza_T

















asked Dec 25 '18 at 13:03









Alireza_TAlireza_T

83




83












  • $begingroup$
    Hi, can you please add some brackets at the first 3 rows for the logarithms? It is abit confusing, but I guess it should be: $lg^2(2n/3)$. Also are you sure that the second row is $T(n)$ instead of $T(2n/3)$?
    $endgroup$
    – Zacky
    Dec 25 '18 at 13:11












  • $begingroup$
    @Zacky Hi! I've added those parentheses and You're right it's T(2n/3) instead of T(n).My bad!
    $endgroup$
    – Alireza_T
    Dec 25 '18 at 13:24












  • $begingroup$
    Also how do you end the recursion, I mean how did your teacher tought you to write afterwards $T(n)$ as a complete sum? I can answer you by how I've learnt, but it might differ.
    $endgroup$
    – Zacky
    Dec 25 '18 at 13:26












  • $begingroup$
    @Zacky My initial thought for this specific example is to reach T(1)! It's kinda like recursion tree problems to me with one branch!
    $endgroup$
    – Alireza_T
    Dec 25 '18 at 13:31




















  • $begingroup$
    Hi, can you please add some brackets at the first 3 rows for the logarithms? It is abit confusing, but I guess it should be: $lg^2(2n/3)$. Also are you sure that the second row is $T(n)$ instead of $T(2n/3)$?
    $endgroup$
    – Zacky
    Dec 25 '18 at 13:11












  • $begingroup$
    @Zacky Hi! I've added those parentheses and You're right it's T(2n/3) instead of T(n).My bad!
    $endgroup$
    – Alireza_T
    Dec 25 '18 at 13:24












  • $begingroup$
    Also how do you end the recursion, I mean how did your teacher tought you to write afterwards $T(n)$ as a complete sum? I can answer you by how I've learnt, but it might differ.
    $endgroup$
    – Zacky
    Dec 25 '18 at 13:26












  • $begingroup$
    @Zacky My initial thought for this specific example is to reach T(1)! It's kinda like recursion tree problems to me with one branch!
    $endgroup$
    – Alireza_T
    Dec 25 '18 at 13:31


















$begingroup$
Hi, can you please add some brackets at the first 3 rows for the logarithms? It is abit confusing, but I guess it should be: $lg^2(2n/3)$. Also are you sure that the second row is $T(n)$ instead of $T(2n/3)$?
$endgroup$
– Zacky
Dec 25 '18 at 13:11






$begingroup$
Hi, can you please add some brackets at the first 3 rows for the logarithms? It is abit confusing, but I guess it should be: $lg^2(2n/3)$. Also are you sure that the second row is $T(n)$ instead of $T(2n/3)$?
$endgroup$
– Zacky
Dec 25 '18 at 13:11














$begingroup$
@Zacky Hi! I've added those parentheses and You're right it's T(2n/3) instead of T(n).My bad!
$endgroup$
– Alireza_T
Dec 25 '18 at 13:24






$begingroup$
@Zacky Hi! I've added those parentheses and You're right it's T(2n/3) instead of T(n).My bad!
$endgroup$
– Alireza_T
Dec 25 '18 at 13:24














$begingroup$
Also how do you end the recursion, I mean how did your teacher tought you to write afterwards $T(n)$ as a complete sum? I can answer you by how I've learnt, but it might differ.
$endgroup$
– Zacky
Dec 25 '18 at 13:26






$begingroup$
Also how do you end the recursion, I mean how did your teacher tought you to write afterwards $T(n)$ as a complete sum? I can answer you by how I've learnt, but it might differ.
$endgroup$
– Zacky
Dec 25 '18 at 13:26














$begingroup$
@Zacky My initial thought for this specific example is to reach T(1)! It's kinda like recursion tree problems to me with one branch!
$endgroup$
– Alireza_T
Dec 25 '18 at 13:31






$begingroup$
@Zacky My initial thought for this specific example is to reach T(1)! It's kinda like recursion tree problems to me with one branch!
$endgroup$
– Alireza_T
Dec 25 '18 at 13:31












2 Answers
2






active

oldest

votes


















3












$begingroup$

$$T(n)=Tleft(frac{2n}{3}right)+lg^2 n$$
$$Tleft(frac{2n}{3}right)=Tleft(frac{2cdotfrac{2n}{3}}{3}right)+lg^2left(frac{2n}{3}right)=Tleft(frac{2^2n}{3^2}right)+lg^2left(frac{2n}{3}right)$$
$$Tleft(frac{2^2n}{3^2}right)=Tleft(frac{2^3n}{3^3}right)+lg^2left(frac{2^2n}{3^2}right)$$



$$dots Tleft(frac{2^{q-1}cdot n}{3^{q-1}}right)= Tleft(frac{2^{q}cdot n}{3^{q}}right)+lg^2 left(frac{2^{q-1}cdot n}{3^{q-1}}right) $$
Now in order to obtain $T(1)$ we consider the limit condition:
$$Tleft(frac{2^qcdot n}{3^q}right)=T(1)Rightarrow left(frac23right)^qn=1Rightarrow n=left(frac{3}{2}right)^qRightarrow log_{frac32}n=q$$
$$Rightarrow T(n)=T(1)+lg^2 n+lg^2left(frac{2n}{3}right)+dots +lg^2 left(frac{2^{q-1}cdot n}{3^{q-1}}right) $$
Now we write the above as a sum. Also the time complexity of $T(1)$ is just $Theta(1)$.
$$T(n)=Theta(1)+sum_{k=0}^{q-1}lg^2left(left(frac{2}{3}right)^knright),quad q=log_{frac32}n$$
Now we have using some basic algebra and properties of the logarithms: $$left(lgleft(left(frac23right)^k cdot nright)right)^2=left(lgleft(frac23right)^k+ lg nright)^2=left(klgleft(frac23right)+ lg nright)^2=$$
$$=k^2 lg^2left(frac23right)+2klgleft(frac23right)lg n+lg^2n$$
$$Rightarrow T(n)=Theta(1)+lg^2left(frac23right)sum_{k=0}^{q-1}k^2+2lgleft(frac23right)lg nsum_{k=0}^{q-1} k +lg^2nsum_{k=0}^{q-1}1$$
$$=Theta(1)+lg^2left(frac23right)frac{(q-1)q(2q-1)}{6}+2lgleft(frac23right)lg nfrac{(q-1)q}{2}+lg^2 n (q-1)$$
Now note that $ displaystyle{q=log_{frac23}{n}=frac{lg n}{lgfrac23}},$ and to obtain the time complexity constants don't matter.
$$T(n)= Theta(1)+c_1 (lg n-1)lg n(2lg n-1)+c_2 lg ncdot (lg n-1)lg n +c_3 lg^2ncdot (lg n-1) $$
$$T(n)=Theta(1)+c_1Theta(lg^3 n)+c_2Theta(lg^3 n)+c_3Theta(lg^3n)=Theta(lg^ 3 n)$$






share|cite|improve this answer











$endgroup$













  • $begingroup$
    Well, thanks a lot.That's exactly what I'm looking for.
    $endgroup$
    – Alireza_T
    Dec 25 '18 at 14:12










  • $begingroup$
    I'm glad that I could help.
    $endgroup$
    – Zacky
    Dec 25 '18 at 14:15



















1












$begingroup$

Look at the generaliyzed recurrence $Tleft(xright)=Tleft(ccdot xright)+left(log_{10}xright)^{2}$ with $0<c<1$.



First of all, $text{Dom}Tleft(xright)subseteqmathbb{R}^{+}$ as $log_{10}x$ existst and $text{Dom}log_{10}xsubseteqmathbb{R}^{+}$. Therefore, define $$U(x)=Tleft(10^{x}right)qquadlog_{10}c=alpha<0$$



Then:



$$Tleft(xright)=Uleft(log_{10}xright)qquad Tleft(cxright)=Uleft(alpha+log_{10}xright)$$



Therefore, for U, the functional equation becomes:



$$Uleft(vright)=Uleft(alpha+vright)+v^{2}$$
Let $v_{1}=-kalpha+beta$ with $betain(0,-alpha]$ and $k$ integer, then



$$Uleft(beta-kalpharight)=Uleft(beta-left(k-1right)alpharight)+left(beta-kalpharight)^{2}$$



Let $v_{2}=kalpha+beta$ with $betain(0,-alpha]$ and $k$ integer, then



$$Uleft(beta+kalpharight)=Uleft(beta+left(k-1right)alpharight)-left(beta-left(k-1right)alpharight)^{2}$$



$$U(v_{1})=sum_{i=1}^{k}left(beta-ialpharight)^{2}+Uleft(betaright)=beta^{2}k-alphabeta k(k+1)+frac{alpha^{2}}{6}k(k+1)(2k+1)+Uleft(betaright)$$



$$U(v_{2})=-sum_{i=0}^{k-1}left(beta-ialpharight)^{2}+Uleft(betaright)=beta^{2}k-alphabeta k(k+1)+frac{alpha^{2}}{6}k(k+1)(2k+1)+Uleft(betaright)$$



$$Uleft(beta-kalpharight)=beta^{2}k-alphabeta k(k+1)+frac{alpha^{2}}{6}k(k+1)(2k+1)+Uleft(betaright)$$



$$Uleft(beta+kalpharight)=-beta^{2}k+alphabeta k(k+1)-frac{alpha^{2}}{6}k(k-1)(2k-1)+Uleft(betaright)$$



Hence, we can define any function $U$ on $(0,-alpha]$ and then extend it with the above formula. Implying $T $ from this is straightforward via the relation $U(x)=Tleft(10^{x}right)$. So in your case, you can have any function $f(x)$ on $left(0,log_{10}frac{2}{3} right]$, extend via the previously decribed relations, and set the function $T$ via $T(x)=U(log_{10}x)$






share|cite|improve this answer









$endgroup$













    Your Answer





    StackExchange.ifUsing("editor", function () {
    return StackExchange.using("mathjaxEditing", function () {
    StackExchange.MarkdownEditor.creationCallbacks.add(function (editor, postfix) {
    StackExchange.mathjaxEditing.prepareWmdForMathJax(editor, postfix, [["$", "$"], ["\\(","\\)"]]);
    });
    });
    }, "mathjax-editing");

    StackExchange.ready(function() {
    var channelOptions = {
    tags: "".split(" "),
    id: "69"
    };
    initTagRenderer("".split(" "), "".split(" "), channelOptions);

    StackExchange.using("externalEditor", function() {
    // Have to fire editor after snippets, if snippets enabled
    if (StackExchange.settings.snippets.snippetsEnabled) {
    StackExchange.using("snippets", function() {
    createEditor();
    });
    }
    else {
    createEditor();
    }
    });

    function createEditor() {
    StackExchange.prepareEditor({
    heartbeatType: 'answer',
    autoActivateHeartbeat: false,
    convertImagesToLinks: true,
    noModals: true,
    showLowRepImageUploadWarning: true,
    reputationToPostImages: 10,
    bindNavPrevention: true,
    postfix: "",
    imageUploader: {
    brandingHtml: "Powered by u003ca class="icon-imgur-white" href="https://imgur.com/"u003eu003c/au003e",
    contentPolicyHtml: "User contributions licensed under u003ca href="https://creativecommons.org/licenses/by-sa/3.0/"u003ecc by-sa 3.0 with attribution requiredu003c/au003e u003ca href="https://stackoverflow.com/legal/content-policy"u003e(content policy)u003c/au003e",
    allowUrls: true
    },
    noCode: true, onDemand: true,
    discardSelector: ".discard-answer"
    ,immediatelyShowMarkdownHelp:true
    });


    }
    });














    draft saved

    draft discarded


















    StackExchange.ready(
    function () {
    StackExchange.openid.initPostLogin('.new-post-login', 'https%3a%2f%2fmath.stackexchange.com%2fquestions%2f3052090%2fhow-to-solve-tn-t2n-3-lg2-n-by-substitution%23new-answer', 'question_page');
    }
    );

    Post as a guest















    Required, but never shown

























    2 Answers
    2






    active

    oldest

    votes








    2 Answers
    2






    active

    oldest

    votes









    active

    oldest

    votes






    active

    oldest

    votes









    3












    $begingroup$

    $$T(n)=Tleft(frac{2n}{3}right)+lg^2 n$$
    $$Tleft(frac{2n}{3}right)=Tleft(frac{2cdotfrac{2n}{3}}{3}right)+lg^2left(frac{2n}{3}right)=Tleft(frac{2^2n}{3^2}right)+lg^2left(frac{2n}{3}right)$$
    $$Tleft(frac{2^2n}{3^2}right)=Tleft(frac{2^3n}{3^3}right)+lg^2left(frac{2^2n}{3^2}right)$$



    $$dots Tleft(frac{2^{q-1}cdot n}{3^{q-1}}right)= Tleft(frac{2^{q}cdot n}{3^{q}}right)+lg^2 left(frac{2^{q-1}cdot n}{3^{q-1}}right) $$
    Now in order to obtain $T(1)$ we consider the limit condition:
    $$Tleft(frac{2^qcdot n}{3^q}right)=T(1)Rightarrow left(frac23right)^qn=1Rightarrow n=left(frac{3}{2}right)^qRightarrow log_{frac32}n=q$$
    $$Rightarrow T(n)=T(1)+lg^2 n+lg^2left(frac{2n}{3}right)+dots +lg^2 left(frac{2^{q-1}cdot n}{3^{q-1}}right) $$
    Now we write the above as a sum. Also the time complexity of $T(1)$ is just $Theta(1)$.
    $$T(n)=Theta(1)+sum_{k=0}^{q-1}lg^2left(left(frac{2}{3}right)^knright),quad q=log_{frac32}n$$
    Now we have using some basic algebra and properties of the logarithms: $$left(lgleft(left(frac23right)^k cdot nright)right)^2=left(lgleft(frac23right)^k+ lg nright)^2=left(klgleft(frac23right)+ lg nright)^2=$$
    $$=k^2 lg^2left(frac23right)+2klgleft(frac23right)lg n+lg^2n$$
    $$Rightarrow T(n)=Theta(1)+lg^2left(frac23right)sum_{k=0}^{q-1}k^2+2lgleft(frac23right)lg nsum_{k=0}^{q-1} k +lg^2nsum_{k=0}^{q-1}1$$
    $$=Theta(1)+lg^2left(frac23right)frac{(q-1)q(2q-1)}{6}+2lgleft(frac23right)lg nfrac{(q-1)q}{2}+lg^2 n (q-1)$$
    Now note that $ displaystyle{q=log_{frac23}{n}=frac{lg n}{lgfrac23}},$ and to obtain the time complexity constants don't matter.
    $$T(n)= Theta(1)+c_1 (lg n-1)lg n(2lg n-1)+c_2 lg ncdot (lg n-1)lg n +c_3 lg^2ncdot (lg n-1) $$
    $$T(n)=Theta(1)+c_1Theta(lg^3 n)+c_2Theta(lg^3 n)+c_3Theta(lg^3n)=Theta(lg^ 3 n)$$






    share|cite|improve this answer











    $endgroup$













    • $begingroup$
      Well, thanks a lot.That's exactly what I'm looking for.
      $endgroup$
      – Alireza_T
      Dec 25 '18 at 14:12










    • $begingroup$
      I'm glad that I could help.
      $endgroup$
      – Zacky
      Dec 25 '18 at 14:15
















    3












    $begingroup$

    $$T(n)=Tleft(frac{2n}{3}right)+lg^2 n$$
    $$Tleft(frac{2n}{3}right)=Tleft(frac{2cdotfrac{2n}{3}}{3}right)+lg^2left(frac{2n}{3}right)=Tleft(frac{2^2n}{3^2}right)+lg^2left(frac{2n}{3}right)$$
    $$Tleft(frac{2^2n}{3^2}right)=Tleft(frac{2^3n}{3^3}right)+lg^2left(frac{2^2n}{3^2}right)$$



    $$dots Tleft(frac{2^{q-1}cdot n}{3^{q-1}}right)= Tleft(frac{2^{q}cdot n}{3^{q}}right)+lg^2 left(frac{2^{q-1}cdot n}{3^{q-1}}right) $$
    Now in order to obtain $T(1)$ we consider the limit condition:
    $$Tleft(frac{2^qcdot n}{3^q}right)=T(1)Rightarrow left(frac23right)^qn=1Rightarrow n=left(frac{3}{2}right)^qRightarrow log_{frac32}n=q$$
    $$Rightarrow T(n)=T(1)+lg^2 n+lg^2left(frac{2n}{3}right)+dots +lg^2 left(frac{2^{q-1}cdot n}{3^{q-1}}right) $$
    Now we write the above as a sum. Also the time complexity of $T(1)$ is just $Theta(1)$.
    $$T(n)=Theta(1)+sum_{k=0}^{q-1}lg^2left(left(frac{2}{3}right)^knright),quad q=log_{frac32}n$$
    Now we have using some basic algebra and properties of the logarithms: $$left(lgleft(left(frac23right)^k cdot nright)right)^2=left(lgleft(frac23right)^k+ lg nright)^2=left(klgleft(frac23right)+ lg nright)^2=$$
    $$=k^2 lg^2left(frac23right)+2klgleft(frac23right)lg n+lg^2n$$
    $$Rightarrow T(n)=Theta(1)+lg^2left(frac23right)sum_{k=0}^{q-1}k^2+2lgleft(frac23right)lg nsum_{k=0}^{q-1} k +lg^2nsum_{k=0}^{q-1}1$$
    $$=Theta(1)+lg^2left(frac23right)frac{(q-1)q(2q-1)}{6}+2lgleft(frac23right)lg nfrac{(q-1)q}{2}+lg^2 n (q-1)$$
    Now note that $ displaystyle{q=log_{frac23}{n}=frac{lg n}{lgfrac23}},$ and to obtain the time complexity constants don't matter.
    $$T(n)= Theta(1)+c_1 (lg n-1)lg n(2lg n-1)+c_2 lg ncdot (lg n-1)lg n +c_3 lg^2ncdot (lg n-1) $$
    $$T(n)=Theta(1)+c_1Theta(lg^3 n)+c_2Theta(lg^3 n)+c_3Theta(lg^3n)=Theta(lg^ 3 n)$$






    share|cite|improve this answer











    $endgroup$













    • $begingroup$
      Well, thanks a lot.That's exactly what I'm looking for.
      $endgroup$
      – Alireza_T
      Dec 25 '18 at 14:12










    • $begingroup$
      I'm glad that I could help.
      $endgroup$
      – Zacky
      Dec 25 '18 at 14:15














    3












    3








    3





    $begingroup$

    $$T(n)=Tleft(frac{2n}{3}right)+lg^2 n$$
    $$Tleft(frac{2n}{3}right)=Tleft(frac{2cdotfrac{2n}{3}}{3}right)+lg^2left(frac{2n}{3}right)=Tleft(frac{2^2n}{3^2}right)+lg^2left(frac{2n}{3}right)$$
    $$Tleft(frac{2^2n}{3^2}right)=Tleft(frac{2^3n}{3^3}right)+lg^2left(frac{2^2n}{3^2}right)$$



    $$dots Tleft(frac{2^{q-1}cdot n}{3^{q-1}}right)= Tleft(frac{2^{q}cdot n}{3^{q}}right)+lg^2 left(frac{2^{q-1}cdot n}{3^{q-1}}right) $$
    Now in order to obtain $T(1)$ we consider the limit condition:
    $$Tleft(frac{2^qcdot n}{3^q}right)=T(1)Rightarrow left(frac23right)^qn=1Rightarrow n=left(frac{3}{2}right)^qRightarrow log_{frac32}n=q$$
    $$Rightarrow T(n)=T(1)+lg^2 n+lg^2left(frac{2n}{3}right)+dots +lg^2 left(frac{2^{q-1}cdot n}{3^{q-1}}right) $$
    Now we write the above as a sum. Also the time complexity of $T(1)$ is just $Theta(1)$.
    $$T(n)=Theta(1)+sum_{k=0}^{q-1}lg^2left(left(frac{2}{3}right)^knright),quad q=log_{frac32}n$$
    Now we have using some basic algebra and properties of the logarithms: $$left(lgleft(left(frac23right)^k cdot nright)right)^2=left(lgleft(frac23right)^k+ lg nright)^2=left(klgleft(frac23right)+ lg nright)^2=$$
    $$=k^2 lg^2left(frac23right)+2klgleft(frac23right)lg n+lg^2n$$
    $$Rightarrow T(n)=Theta(1)+lg^2left(frac23right)sum_{k=0}^{q-1}k^2+2lgleft(frac23right)lg nsum_{k=0}^{q-1} k +lg^2nsum_{k=0}^{q-1}1$$
    $$=Theta(1)+lg^2left(frac23right)frac{(q-1)q(2q-1)}{6}+2lgleft(frac23right)lg nfrac{(q-1)q}{2}+lg^2 n (q-1)$$
    Now note that $ displaystyle{q=log_{frac23}{n}=frac{lg n}{lgfrac23}},$ and to obtain the time complexity constants don't matter.
    $$T(n)= Theta(1)+c_1 (lg n-1)lg n(2lg n-1)+c_2 lg ncdot (lg n-1)lg n +c_3 lg^2ncdot (lg n-1) $$
    $$T(n)=Theta(1)+c_1Theta(lg^3 n)+c_2Theta(lg^3 n)+c_3Theta(lg^3n)=Theta(lg^ 3 n)$$






    share|cite|improve this answer











    $endgroup$



    $$T(n)=Tleft(frac{2n}{3}right)+lg^2 n$$
    $$Tleft(frac{2n}{3}right)=Tleft(frac{2cdotfrac{2n}{3}}{3}right)+lg^2left(frac{2n}{3}right)=Tleft(frac{2^2n}{3^2}right)+lg^2left(frac{2n}{3}right)$$
    $$Tleft(frac{2^2n}{3^2}right)=Tleft(frac{2^3n}{3^3}right)+lg^2left(frac{2^2n}{3^2}right)$$



    $$dots Tleft(frac{2^{q-1}cdot n}{3^{q-1}}right)= Tleft(frac{2^{q}cdot n}{3^{q}}right)+lg^2 left(frac{2^{q-1}cdot n}{3^{q-1}}right) $$
    Now in order to obtain $T(1)$ we consider the limit condition:
    $$Tleft(frac{2^qcdot n}{3^q}right)=T(1)Rightarrow left(frac23right)^qn=1Rightarrow n=left(frac{3}{2}right)^qRightarrow log_{frac32}n=q$$
    $$Rightarrow T(n)=T(1)+lg^2 n+lg^2left(frac{2n}{3}right)+dots +lg^2 left(frac{2^{q-1}cdot n}{3^{q-1}}right) $$
    Now we write the above as a sum. Also the time complexity of $T(1)$ is just $Theta(1)$.
    $$T(n)=Theta(1)+sum_{k=0}^{q-1}lg^2left(left(frac{2}{3}right)^knright),quad q=log_{frac32}n$$
    Now we have using some basic algebra and properties of the logarithms: $$left(lgleft(left(frac23right)^k cdot nright)right)^2=left(lgleft(frac23right)^k+ lg nright)^2=left(klgleft(frac23right)+ lg nright)^2=$$
    $$=k^2 lg^2left(frac23right)+2klgleft(frac23right)lg n+lg^2n$$
    $$Rightarrow T(n)=Theta(1)+lg^2left(frac23right)sum_{k=0}^{q-1}k^2+2lgleft(frac23right)lg nsum_{k=0}^{q-1} k +lg^2nsum_{k=0}^{q-1}1$$
    $$=Theta(1)+lg^2left(frac23right)frac{(q-1)q(2q-1)}{6}+2lgleft(frac23right)lg nfrac{(q-1)q}{2}+lg^2 n (q-1)$$
    Now note that $ displaystyle{q=log_{frac23}{n}=frac{lg n}{lgfrac23}},$ and to obtain the time complexity constants don't matter.
    $$T(n)= Theta(1)+c_1 (lg n-1)lg n(2lg n-1)+c_2 lg ncdot (lg n-1)lg n +c_3 lg^2ncdot (lg n-1) $$
    $$T(n)=Theta(1)+c_1Theta(lg^3 n)+c_2Theta(lg^3 n)+c_3Theta(lg^3n)=Theta(lg^ 3 n)$$







    share|cite|improve this answer














    share|cite|improve this answer



    share|cite|improve this answer








    edited Dec 26 '18 at 12:13

























    answered Dec 25 '18 at 14:07









    ZackyZacky

    7,62011061




    7,62011061












    • $begingroup$
      Well, thanks a lot.That's exactly what I'm looking for.
      $endgroup$
      – Alireza_T
      Dec 25 '18 at 14:12










    • $begingroup$
      I'm glad that I could help.
      $endgroup$
      – Zacky
      Dec 25 '18 at 14:15


















    • $begingroup$
      Well, thanks a lot.That's exactly what I'm looking for.
      $endgroup$
      – Alireza_T
      Dec 25 '18 at 14:12










    • $begingroup$
      I'm glad that I could help.
      $endgroup$
      – Zacky
      Dec 25 '18 at 14:15
















    $begingroup$
    Well, thanks a lot.That's exactly what I'm looking for.
    $endgroup$
    – Alireza_T
    Dec 25 '18 at 14:12




    $begingroup$
    Well, thanks a lot.That's exactly what I'm looking for.
    $endgroup$
    – Alireza_T
    Dec 25 '18 at 14:12












    $begingroup$
    I'm glad that I could help.
    $endgroup$
    – Zacky
    Dec 25 '18 at 14:15




    $begingroup$
    I'm glad that I could help.
    $endgroup$
    – Zacky
    Dec 25 '18 at 14:15











    1












    $begingroup$

    Look at the generaliyzed recurrence $Tleft(xright)=Tleft(ccdot xright)+left(log_{10}xright)^{2}$ with $0<c<1$.



    First of all, $text{Dom}Tleft(xright)subseteqmathbb{R}^{+}$ as $log_{10}x$ existst and $text{Dom}log_{10}xsubseteqmathbb{R}^{+}$. Therefore, define $$U(x)=Tleft(10^{x}right)qquadlog_{10}c=alpha<0$$



    Then:



    $$Tleft(xright)=Uleft(log_{10}xright)qquad Tleft(cxright)=Uleft(alpha+log_{10}xright)$$



    Therefore, for U, the functional equation becomes:



    $$Uleft(vright)=Uleft(alpha+vright)+v^{2}$$
    Let $v_{1}=-kalpha+beta$ with $betain(0,-alpha]$ and $k$ integer, then



    $$Uleft(beta-kalpharight)=Uleft(beta-left(k-1right)alpharight)+left(beta-kalpharight)^{2}$$



    Let $v_{2}=kalpha+beta$ with $betain(0,-alpha]$ and $k$ integer, then



    $$Uleft(beta+kalpharight)=Uleft(beta+left(k-1right)alpharight)-left(beta-left(k-1right)alpharight)^{2}$$



    $$U(v_{1})=sum_{i=1}^{k}left(beta-ialpharight)^{2}+Uleft(betaright)=beta^{2}k-alphabeta k(k+1)+frac{alpha^{2}}{6}k(k+1)(2k+1)+Uleft(betaright)$$



    $$U(v_{2})=-sum_{i=0}^{k-1}left(beta-ialpharight)^{2}+Uleft(betaright)=beta^{2}k-alphabeta k(k+1)+frac{alpha^{2}}{6}k(k+1)(2k+1)+Uleft(betaright)$$



    $$Uleft(beta-kalpharight)=beta^{2}k-alphabeta k(k+1)+frac{alpha^{2}}{6}k(k+1)(2k+1)+Uleft(betaright)$$



    $$Uleft(beta+kalpharight)=-beta^{2}k+alphabeta k(k+1)-frac{alpha^{2}}{6}k(k-1)(2k-1)+Uleft(betaright)$$



    Hence, we can define any function $U$ on $(0,-alpha]$ and then extend it with the above formula. Implying $T $ from this is straightforward via the relation $U(x)=Tleft(10^{x}right)$. So in your case, you can have any function $f(x)$ on $left(0,log_{10}frac{2}{3} right]$, extend via the previously decribed relations, and set the function $T$ via $T(x)=U(log_{10}x)$






    share|cite|improve this answer









    $endgroup$


















      1












      $begingroup$

      Look at the generaliyzed recurrence $Tleft(xright)=Tleft(ccdot xright)+left(log_{10}xright)^{2}$ with $0<c<1$.



      First of all, $text{Dom}Tleft(xright)subseteqmathbb{R}^{+}$ as $log_{10}x$ existst and $text{Dom}log_{10}xsubseteqmathbb{R}^{+}$. Therefore, define $$U(x)=Tleft(10^{x}right)qquadlog_{10}c=alpha<0$$



      Then:



      $$Tleft(xright)=Uleft(log_{10}xright)qquad Tleft(cxright)=Uleft(alpha+log_{10}xright)$$



      Therefore, for U, the functional equation becomes:



      $$Uleft(vright)=Uleft(alpha+vright)+v^{2}$$
      Let $v_{1}=-kalpha+beta$ with $betain(0,-alpha]$ and $k$ integer, then



      $$Uleft(beta-kalpharight)=Uleft(beta-left(k-1right)alpharight)+left(beta-kalpharight)^{2}$$



      Let $v_{2}=kalpha+beta$ with $betain(0,-alpha]$ and $k$ integer, then



      $$Uleft(beta+kalpharight)=Uleft(beta+left(k-1right)alpharight)-left(beta-left(k-1right)alpharight)^{2}$$



      $$U(v_{1})=sum_{i=1}^{k}left(beta-ialpharight)^{2}+Uleft(betaright)=beta^{2}k-alphabeta k(k+1)+frac{alpha^{2}}{6}k(k+1)(2k+1)+Uleft(betaright)$$



      $$U(v_{2})=-sum_{i=0}^{k-1}left(beta-ialpharight)^{2}+Uleft(betaright)=beta^{2}k-alphabeta k(k+1)+frac{alpha^{2}}{6}k(k+1)(2k+1)+Uleft(betaright)$$



      $$Uleft(beta-kalpharight)=beta^{2}k-alphabeta k(k+1)+frac{alpha^{2}}{6}k(k+1)(2k+1)+Uleft(betaright)$$



      $$Uleft(beta+kalpharight)=-beta^{2}k+alphabeta k(k+1)-frac{alpha^{2}}{6}k(k-1)(2k-1)+Uleft(betaright)$$



      Hence, we can define any function $U$ on $(0,-alpha]$ and then extend it with the above formula. Implying $T $ from this is straightforward via the relation $U(x)=Tleft(10^{x}right)$. So in your case, you can have any function $f(x)$ on $left(0,log_{10}frac{2}{3} right]$, extend via the previously decribed relations, and set the function $T$ via $T(x)=U(log_{10}x)$






      share|cite|improve this answer









      $endgroup$
















        1












        1








        1





        $begingroup$

        Look at the generaliyzed recurrence $Tleft(xright)=Tleft(ccdot xright)+left(log_{10}xright)^{2}$ with $0<c<1$.



        First of all, $text{Dom}Tleft(xright)subseteqmathbb{R}^{+}$ as $log_{10}x$ existst and $text{Dom}log_{10}xsubseteqmathbb{R}^{+}$. Therefore, define $$U(x)=Tleft(10^{x}right)qquadlog_{10}c=alpha<0$$



        Then:



        $$Tleft(xright)=Uleft(log_{10}xright)qquad Tleft(cxright)=Uleft(alpha+log_{10}xright)$$



        Therefore, for U, the functional equation becomes:



        $$Uleft(vright)=Uleft(alpha+vright)+v^{2}$$
        Let $v_{1}=-kalpha+beta$ with $betain(0,-alpha]$ and $k$ integer, then



        $$Uleft(beta-kalpharight)=Uleft(beta-left(k-1right)alpharight)+left(beta-kalpharight)^{2}$$



        Let $v_{2}=kalpha+beta$ with $betain(0,-alpha]$ and $k$ integer, then



        $$Uleft(beta+kalpharight)=Uleft(beta+left(k-1right)alpharight)-left(beta-left(k-1right)alpharight)^{2}$$



        $$U(v_{1})=sum_{i=1}^{k}left(beta-ialpharight)^{2}+Uleft(betaright)=beta^{2}k-alphabeta k(k+1)+frac{alpha^{2}}{6}k(k+1)(2k+1)+Uleft(betaright)$$



        $$U(v_{2})=-sum_{i=0}^{k-1}left(beta-ialpharight)^{2}+Uleft(betaright)=beta^{2}k-alphabeta k(k+1)+frac{alpha^{2}}{6}k(k+1)(2k+1)+Uleft(betaright)$$



        $$Uleft(beta-kalpharight)=beta^{2}k-alphabeta k(k+1)+frac{alpha^{2}}{6}k(k+1)(2k+1)+Uleft(betaright)$$



        $$Uleft(beta+kalpharight)=-beta^{2}k+alphabeta k(k+1)-frac{alpha^{2}}{6}k(k-1)(2k-1)+Uleft(betaright)$$



        Hence, we can define any function $U$ on $(0,-alpha]$ and then extend it with the above formula. Implying $T $ from this is straightforward via the relation $U(x)=Tleft(10^{x}right)$. So in your case, you can have any function $f(x)$ on $left(0,log_{10}frac{2}{3} right]$, extend via the previously decribed relations, and set the function $T$ via $T(x)=U(log_{10}x)$






        share|cite|improve this answer









        $endgroup$



        Look at the generaliyzed recurrence $Tleft(xright)=Tleft(ccdot xright)+left(log_{10}xright)^{2}$ with $0<c<1$.



        First of all, $text{Dom}Tleft(xright)subseteqmathbb{R}^{+}$ as $log_{10}x$ existst and $text{Dom}log_{10}xsubseteqmathbb{R}^{+}$. Therefore, define $$U(x)=Tleft(10^{x}right)qquadlog_{10}c=alpha<0$$



        Then:



        $$Tleft(xright)=Uleft(log_{10}xright)qquad Tleft(cxright)=Uleft(alpha+log_{10}xright)$$



        Therefore, for U, the functional equation becomes:



        $$Uleft(vright)=Uleft(alpha+vright)+v^{2}$$
        Let $v_{1}=-kalpha+beta$ with $betain(0,-alpha]$ and $k$ integer, then



        $$Uleft(beta-kalpharight)=Uleft(beta-left(k-1right)alpharight)+left(beta-kalpharight)^{2}$$



        Let $v_{2}=kalpha+beta$ with $betain(0,-alpha]$ and $k$ integer, then



        $$Uleft(beta+kalpharight)=Uleft(beta+left(k-1right)alpharight)-left(beta-left(k-1right)alpharight)^{2}$$



        $$U(v_{1})=sum_{i=1}^{k}left(beta-ialpharight)^{2}+Uleft(betaright)=beta^{2}k-alphabeta k(k+1)+frac{alpha^{2}}{6}k(k+1)(2k+1)+Uleft(betaright)$$



        $$U(v_{2})=-sum_{i=0}^{k-1}left(beta-ialpharight)^{2}+Uleft(betaright)=beta^{2}k-alphabeta k(k+1)+frac{alpha^{2}}{6}k(k+1)(2k+1)+Uleft(betaright)$$



        $$Uleft(beta-kalpharight)=beta^{2}k-alphabeta k(k+1)+frac{alpha^{2}}{6}k(k+1)(2k+1)+Uleft(betaright)$$



        $$Uleft(beta+kalpharight)=-beta^{2}k+alphabeta k(k+1)-frac{alpha^{2}}{6}k(k-1)(2k-1)+Uleft(betaright)$$



        Hence, we can define any function $U$ on $(0,-alpha]$ and then extend it with the above formula. Implying $T $ from this is straightforward via the relation $U(x)=Tleft(10^{x}right)$. So in your case, you can have any function $f(x)$ on $left(0,log_{10}frac{2}{3} right]$, extend via the previously decribed relations, and set the function $T$ via $T(x)=U(log_{10}x)$







        share|cite|improve this answer












        share|cite|improve this answer



        share|cite|improve this answer










        answered Dec 25 '18 at 13:55









        Ákos SomogyiÁkos Somogyi

        1,557417




        1,557417






























            draft saved

            draft discarded




















































            Thanks for contributing an answer to Mathematics Stack Exchange!


            • Please be sure to answer the question. Provide details and share your research!

            But avoid



            • Asking for help, clarification, or responding to other answers.

            • Making statements based on opinion; back them up with references or personal experience.


            Use MathJax to format equations. MathJax reference.


            To learn more, see our tips on writing great answers.




            draft saved


            draft discarded














            StackExchange.ready(
            function () {
            StackExchange.openid.initPostLogin('.new-post-login', 'https%3a%2f%2fmath.stackexchange.com%2fquestions%2f3052090%2fhow-to-solve-tn-t2n-3-lg2-n-by-substitution%23new-answer', 'question_page');
            }
            );

            Post as a guest















            Required, but never shown





















































            Required, but never shown














            Required, but never shown












            Required, but never shown







            Required, but never shown

































            Required, but never shown














            Required, but never shown












            Required, but never shown







            Required, but never shown







            Popular posts from this blog

            To store a contact into the json file from server.js file using a class in NodeJS

            Redirect URL with Chrome Remote Debugging Android Devices

            Dieringhausen